Classify the following triangle. Check all that apply.

Classify The Following Triangle. Check All That Apply.

Answers

Answer 1

Answer:

B.

F.

Step-by-step explanation:

Look at the angle. It is obtuse so the triangle is obtuse.

The sides are congruent so it is an isosceles.

Answer 2

Answer:

B,F

Step-by-step explanation:

sides are equal.

One angle=104°>90°

so itis an isosceles obtuse triangle.


Related Questions

BEST ANSWER GETS TO CHOOSE BRAINLESST OR FOLLOW!​

Answers

Answer:13 is>

14 is =

15 is >

16 is<

17 is<

18 is =

Step-by-step explanation:

I just know plz mark brainliest

a) the quadratic x^2-4x-21 can be written in the form (x+a)^2+b find the values of and and b

b) hence, or otherwise, solve the equation x^2-4x-21=0

Answers

Part a. A=-2. B=-25
Part b solutions; x=7 x=-3

Is the relation a function?
{(-6, -1), (5,-1), (0, -1), (-2, -1), (3, -1)}

Answers

Answer:

yes it is a function

Step-by-step explanation:

For that no two x values are the same with different y values

Please Help me it’s geometry

Answers

Answer:

Option C.

Step-by-step explanation:

Given: In [tex]\Delta OPQ,m\angle O=107^{\circ},m\angle P=28^{\circ} [/tex].

In [tex]\Delta OPQ[/tex],

[tex]m\angle O+m\angle P+m\angle Q=180^{\circ}[/tex]    (Angle sum property)

[tex]107^{\circ}+28^{\circ}+m\angle Q=180^{\circ}[/tex]

[tex]135^{\circ}+m\angle Q=180^{\circ}[/tex]

[tex]m\angle Q=180^{\circ}-135^{\circ}[/tex]

[tex]m\angle Q=45^{\circ}[/tex]

Now,

[tex]m\angle O>m\angle Q>m\angle P[/tex]

In a triangle, the greatest angle has largest opposite side and smallest angle has smallest opposite side. So, we conclude that

[tex]PQ>OP>QO[/tex]

Therefore, the correct option is C.

If f(x) = 2x + 1 and g(x) = x-2 what is the value of f(g(f(3)))? This is so confusing plz help me will mark brainliest A) 1 B) 3 C) 5 D) 7 E) 11

Answers

Answer:

11

Step-by-step explanation:

f(3) = 2*3+1 = 6+1 = 7

Then find g(7)

g(7) = 7-2 = 5

Then find f(5)

f(5) = 2*5 +1 = 10+1 = 11

f(g(f(3))) = 11

Answer:

11

Step-by-step explanation:

f(x)=2x + 1

f(3)= 2.3 + 1

f(3)=7

f(g(f(3))) = f(g(7))

Now, we have to find g(7)

g(x)=x -2

g(7)= 7 -2

g(7)=5

f(g(f(3))) = f(g(7)) = f(5)

now we have to find f(5)

f(x)=2x + 1

f(5)=2.5 + 1

f(5)=11

Hope this helps ^-^

HELP ASAP it says my questions should be at least 20 characters long so what I’m saying right now doesn’t mean anything.

Answers

Answer: See pic above for answer. I got it from Photomath

Plz put brainliest

What is -6 divided by -1.2?

Answers

Answer:

5.

Step-by-step explanation:

This is the same as saying:

-6/(-6/5).

To solve the equation, we multiply by the reciprocal and simplify. In this case, we multiply both sides of the equation by -5/6.

-6 * (-5/6) = 5.

The answer must be positive, as two negatives divided equal a positive.

I hope this helps!

The solution of expression after divide is,

⇒ 5

We have to given that;

⇒ - 6 is divided by - 1.2

Now, We can simplify as,

⇒ - 6 is divided by - 1.2

It can be written as,

⇒ - 6 ÷ - 1.2

⇒ - 6 / - 1.2

⇒ 6 / 1.2

⇒ 60 / 12

⇒ 5

Thus, The solution of expression after divide is,

⇒ 5

Learn more about the divide visit:

https://brainly.com/question/28119824

#SPJ6

PLEASE HELP WILL GIVE BRAINLIEST AND 20 POINTS

solve this system of equations

3y-5x=12
y=1/3x

Answers

Answer:

x=-3

y=-1

(-3,-1)

Answer:

(-3, -1).

Step-by-step explanation:

I have attached the work to your problem.

Please see the attachment below.

I hope this helps!

Becky made 4 quarts of chicken noodle soup in a big pot, then served 112-cup bowls of soup to 6 people. How much soup is left in the pot?

Answers

Answer:

1qt 3c

Step-by-step explanation:

I need help with is question ASAP please

Answers

The y-intercept looks like it's 0 and the slop looks like 6/2 or 3 or maybe 7/2

Line I and h intersect at what point

Answers

Answer:

please show where the lines are graphed in a picture

Step-by-step explanation:

Pls help me on this................

Answers

Answer:

Step-by-step explanation:


[tex]( {2}^{ - 1} + {3}^{ - 1} )^{2} [/tex]
solve.

will give the brainliest​

Answers

Answer:

25/36

Step-by-step explanation:

(2^-1 + 3^-1)^2

(1/2 + 1/3)^2

(5/6)^2 = 25/36

25/36 or .694

2^-1= 1/2
3^-1=1/3

Add the together
5/6 square it and you get 25/36

evaluate the following expression -7x(7+9)

Answers

Answer:

-112

Step-by-step explanation:

Add

-7(7+9)

Multiply

-7x16

Wallah! You have the answer

-112

[tex]\text{Simplify the expression:}\\\\-7x(7+9)\\\\\text{Use the distributive property}\\\\-49x-63x\\\\\text{Combine like terms}\\\\\boxed{-112x}[/tex]

Micheal has 1/2 of a yard of fabric to make book covers. Each book cover is made from 1/8 of a yard of fabric. How many book covers can Micheal make?

Answers

Answer:

4

Step-by-step explanation:

[tex]\frac{1}{2}[/tex] ÷ [tex]\frac{1}{8}[/tex] = 4

A. 256
B. 265
C. 297
D. 279​

Answers

Answer:

A.

Step-by-step explanation:

So calculate one of the triangles.

[tex]\frac{12*8}{2} =48[/tex]

48 * 4 = 192

192 + 64 = 256

14. Roger is on a playground swing, and he is swinging back and forth in such a way that the height, h, in feet, of the swing off the ground is given by the equation h=3cos(3π/2t) +5, where t is in seconds. How many seconds elapses between two consecutive times that the swing is at its maximum height?​

Answers

Answer:

The time [tex]t = \frac{3}{2}[/tex]  seconds elapses between two consecutive times that the swing is at its maximum height  'h' = 2

 Step-by-step explanation:

Explanation:-

Step(i):-

Given function [tex]h(t) = 3 cos (\frac{3\pi }{2 t} ) +5[/tex] ....(i)

By using derivative formulas

[tex]\frac{d cosx }{d x} = -sinx[/tex]

[tex]\frac{d x^{n} }{d x} = n x^{n-1}[/tex]

[tex]\frac{d t^{-1} }{d x} = -1 t^{-1-1} = - t^{-2} = \frac{-1}{t^{2} }[/tex]

Step(ii):-

Differentiating equation(i) with respective to 't'

[tex]h^{l} (t) = 3(-sin(\frac{3\pi }{2t})\frac{d}{dt} (\frac{3\pi }{2t } )+0[/tex]  ...(ii)

[tex]h^{l} (t) = 3(-sin(\frac{3\pi }{2t})(\frac{-3\pi }{2t^{2} } )+0[/tex]

Equating zero

[tex]h^{l} (t) = 3(-sin(\frac{3\pi }{2t})(\frac{-3\pi }{2t^{2} } )=0[/tex]

[tex]3(-sin(\frac{3\pi }{2t})(\frac{-3\pi }{2t^{2} } ) = 0[/tex]

on simplification , we get

[tex](sin(\frac{3\pi }{2t}) = 0[/tex]

now we use formulas

sin 0 = 0 and sinπ = 0

General solution

[tex](sin(\frac{3\pi }{2t}) = sin\pi[/tex]

[tex](\frac{3\pi }{2t}) = \pi[/tex]

Cancellation 'π' on both sides, we get

[tex]3 = 2 t[/tex]

Dividing '2' on both sides , we get

[tex]t = \frac{3}{2}[/tex]

Again differentiating with respective to 't' , we get  

[tex]h^{ll} (t) = 3(-cos(\frac{3\pi }{2t})(\frac{-3\pi }{2t^{2} } )+ (-3)(-sin(\frac{3\pi }{2t} )(\frac{6\pi }{2t^{3} }[/tex]

Put t= 3/2 and simplification

[tex]h^{ll} (t) < 0[/tex]

The maximum height

                   [tex]h(t) = 3 cos (\frac{3\pi }{2 t} ) +5[/tex]

               [tex]h(\frac{3}{2} ) = 3 cos (\frac{3\pi }{2(\frac{3}{2} )} )+5[/tex]

              [tex]h(\frac{3}{2} ) = 3 cos (\pi )+5 = -3+5 =2[/tex]

[tex]t = \frac{3}{2}[/tex]  seconds elapses between two consecutive times that the swing is at its maximum height  'h' = 2

Conclusion:-

The time [tex]t = \frac{3}{2}[/tex]  seconds elapses between two consecutive times that the swing is at its maximum height  'h' = 2

 

 

A few years ago, a country consumed 19,400,000 barrels of oil per day. That same year, there were about 119,000,000 households in that country. A barrel of oil is 42 U.S. gallons. Estimate how many gallons each household in the country consumed each day that year by rounding down all three numbers. (Round 19,400,000 barrels to 19,000,000 barrels, 119,000,000 households to 115,000,000 households, and 42 gallons to 40 gallons. Round your answer to two decimal places.)

_________ gal/household/day

Calculate the answer without using estimation and compare the two results. (Round your answer to two decimal places.)
___________gal/household/day

Answers

Following are the calculation to fill in the blank:

For Part a)

Each home in this town consumed

[tex]\to 40 \times \frac{19000000}{115000000}\\\\\to 40 \times \frac{19}{115}\\\\\to 40 \times 0.165\\\\\to 6.6 \ \text{gallons of oil per day}[/tex]

For Part b)

Without completing the sentence,

[tex]\text{Total consumption = 19200000 barrels}\\\\\text{Total number of households = 118000000}[/tex]

As a result, each household consumed.

[tex]\to \frac{19200000}{118000000} \\\\\to \frac{192}{1180} \ (barrels)\\\\[/tex]

In the given scenario, each barrel includes 42 gallons of oil. Therefore, every household consumed [tex]42 \times \frac{192}{1180} = 6.83 \ gallons[/tex] oil each day.

Learn more:

brainly.com/question/14798687

Hence, each household will consume 6.60 gallons per day.

Total Households of that country =115000000 (approx.)

Total barrels used per day =19000000 (approx.)

Given 1 barrel = 40 gallons (approx.)

So total gallons used per day = 19000000 x 40 =760000000

So each household will consume gallons per day = total gallons used per day / Total household of that country

=[tex]=\frac{760000000}{ 11500000}[/tex]

=6.60 gallons/household/day

So each household will consume 6.60 gallons per day.

Learn More: https://brainly.com/question/12272570

If g(x) = -4x + 5, find g(2x-1).

Answers

Answer:

- 8x + 9

Step-by-step explanation:

To evaluate g(2x - 1) substitute x = 2x - 1 into g(x), that is

g(2x - 1)

= - 4(2x - 1) + 5 ← distribute parenthesis and simplify

= - 8x + 4 + 5

= - 8x + 9

Answer:

= - 8x + 9

Step-by-step explanation:

= - 8x + 4 + 5

= - 8x + 9

Jake is earning money for a new TV. So far, he has put $54.29, $87.20, and $21.44 into his savings account. How much money has Jake saved?

Answers

add all the values together

54.29+87.20=141.49

141.49+21.44=162.93

Answer:

$162.93

Step-by-step explanation:

Do 54.29 + 87.20 + 21.44 and get 162.93 than put this $ in front of you answer  then you have $162.93



A new car is purchased for 23400 dollars. The value of the car depreciates at 11.5% per year. To the nearest year, how long will it be until the value of the car is 12700 dollars?

Answers

Answer: 5 years

Step-by-step explanation:

Using same formula as for Compound Interest:

12700 = 23400 (1-.115)^t

12700 = 23400 x 0.885^t

0.885^t = 127/234

Convert decimal into fraction:

177/200^t = 127/234

Take logarithm of both sides of thr equation:

t = log 177/200 (127/234)

t = 5.00242

What is the midpoint of the segment below? (2,3)(-3,-2)

Answers

Answer:

(-0.5, 0.5)

Step-by-step explanation:

If those two points are endpoints, just use the midpoint formula.

(-0.5, 0.5)

Answer:

(-1/2,1/2)

Step-by-step explanation:

To find the midpoint, add the x coordinates and divide by 2

(2+-3)/2 = -1/2

add the y coordinates and divide by 2

(3+-2)/2 = 1/2

The midpoint is (-1/2,1/2)

Helppppp meeeee answer thissssssss

Answers

Answer:

(-1.025)^3 x (-1.025)^2=(- 1.025)^5

(-y)^3 x (-y)^2 x (-y)= y^6

5= odd, - stays

6= even, - becomes +

Answer:

5) (-1.025)^3* (-1.025)^2 = (-1.025)^5 = (-41/40)^5

10) -y^3 * -y^2 * -y

Multiplying an odd number of negative terms makes the product negative

= - y^3 * y^2 * y

= - y^6

Which equation represents a line parallel to the line whose equations is -2x + 3y =
-4 and passes through the point (1,3)?

Answers

Answer:

2.  y - 3 = 2/3 (x - 1).

Step-by-step explanation:

-2x + 3y = -4

3y = 2x - 4

y = 2/3 x - 4/3 - so the slope is  2/3.

The slope of a line parallel to it is also 2/3.

It also passes through the point (1, 3).

Using the point-slope form of  a line:

y - y1 = m(x - x1) where m = the slope and (x1, y1) is a point on the line, we have:

y - 3 = 2/3 (x - 1)  <--- is the required equation.

Answer:

2

Step-by-step explanation:

parallel lines have same slope with different intercept

y= mx+b

m is going to be same with different b

so the given function is:

-2x+3y=-43y= 2x-4y= 2/3x - 4/3

Given options:

1. y-3= - 2/3(x-1)  ⇒ y= -2/3x +3 + 2/3 ⇒ y= -2/3x +11/3

it has different slope, so is not parallel

2. y-3= 2/3(x-1) ⇒ y= 2/3x+3-2/3 ⇒ y= 2/3x + 7/3

it has same slope, so is parallelit should be passing through point (1,3)3= 2/3+7/3 ⇒ 3=3, yes it does

3. y-3= -2/3(x+1) ⇒ y= - 2/3x +3- 2/3 ⇒ y= -2/3x + 7/3

it has different slope, is not parallel

4. y-3= 2/3(x+1) ⇒ y= 2/3x +3+ 2/3 ⇒ y= 2/3x +11/3

it has same slope, so is parallelit should be passing through point (1,3)3= 2/3+11/3 ⇒ 3≠13/3, no it doesn't

Find the solution(s) to x^2- 14x + 49 = 0.
O A. x=-2 and x = 7
B. x= -1 and x = 14
C. x= 7 only
D. x = 7 and x = -7

Answers

Answer:

[tex] \boxed{C. \: x = 7 \: only} [/tex]

Step-by-step explanation:

[tex] = > {x}^{2} - 14x + 49 = 0 \\ \\ = > {x}^{2} - (7 + 7)x + 49 = 0 \\ \\ = > {x}^{2} - 7x - 7x + 49 = 0 \\ \\ = > x(x - 7) - 7(x - 7) = 0 \\ \\ = > (x - 7)(x - 7) = 0 \\ \\ = > {(x - 7)}^{2} = 0 \\ \\ = > x - 7 = 0 \\ \\ = > x = 7[/tex]

Find the value of x in the triangle pair below

Answers

Answer:

x=16.97056274... or 12[tex]\sqrt{2}[/tex]

Step-by-step explanation:

the hypotenuse of a 45, 45, 90 triangle is the length of the leg times the square root of 2

please help as soon as possible
I WILL MARK YOU AS BRAINLIEST ​

Answers

Answer:

Last choice

Step-by-step explanation: Subtract 11 and then divide by 3. A would be greater than -2 and last choice shows that

00:00
Brice is finding the sum of 468 and 241 by breaking it into smaller problems
He uses place value and finds the sums of the hundreds, tens, and ones.
What is the sum of the tens? Enter your answer in the box.
1​

Answers

468+241=709 so 0 is in the tenth place because 6+4= 10 and one gets carried over to the 4+2

Tiffany cells 2 kinds of homemade tomato sauce

Answers

Answer:

b

Step-by-step explanation:

Let x represent the number of quarts of Tuscan sauce and

y represents the number of quarts of marinara sauce Tiffany makes.

A quart of Tuscan sauce requires 6 tomatoes and 1 cup of oil

x quarts requires 6x tomatoes and 1x cups of oil

A quart of her marinara sauce requires 5 tomatoes and 1.25 cups of oil

y quarts requires 5y tomatoes and 1.25 y cups of oil

She has 45 tomatoes and 10 cups of oil on hand.

So the constraints are

6x+5y≤45

1x+1.25y≤10

x>=0 and y>=0

After drawing a diagram of what's required to make 1 quart of Tuscan and 1 quart of Marinara sauce, you can more clearly choose answer choice (B) 6x + 5y < (or equal to) 45, x + 5/4y < (or equal to) 10, x>(or equal to)0, y>(or equal to)0.

please mark brainliest :)

the answer is B !!! hope it’s correct

What is the equation in slope-intercept form of a line with slope of 5 and y-intercept of 3?

Answers

y=5x+3

Because the starting value(y intercept is 3) and the slope(x) is 5

Other Questions
Which is the most accurate score card for the second period of revolution? The Americans were clearly in the lead with more victories. Both American and British had won engagements. The British were clearly in the lead with more victories. The French had claimed new lands causing the British to lose territory. a one-two paragraph summary on the Running Queries and Reports tutorials. Apply critical thinking and an academic writing style that demonstrates your understanding of the difference between a Microsoft Access database and an Excel spreadsheet by comparing the features of each and when they would be used as personal computer applications if applicable. The Latin root "omni" means all or every. How does this help you to determine what third person omniscient point of view means? Omniscient point of view means the perpective of limited characters. Omniscient point of view means the perspective of all of the narrator's thoughts. Omniscient point of view means the perspective of every setting in the story. Omniscient point of view means the perspective of all or every character. You plan to conduct a marketing experiment in which students are to taste one of two different brands of soft drink. Their task is to correctly identify the brand tasted. You select a random sample of 200 students and assume that the students have no ability to distinguish between the two brands. (Hint: if an individual has no ability to distinguish between the two soft drinks, then each brand is equally likely to be selected.)(a) What is the probability that the sample will have between 50% and 60% of the identification correct?(b) The probability is 90% that the sample percentage contained within what symmetrical limits of the population percentage?(c) What is the probability that the sample percentage of correct identifications is greater than 65%? F Choose the correct form of the verb.25. T (salen, sales) con ellos.26. Qu (traen, traemos) nosotros?27. Ustedes (traen, traemos) mucho, no?28. Ella (haces, hace) mucho trabajo.29. Nosotros (salgo, salimos) a tiempo.30. Ellos (hacen, hacemos) el viaje juntos.Give the A stationary speed gun emits a microwave beam at 2.10*10^10Hz. It reflects off a car and returns 1030 Hz higher. What is the speed of the car? plz help me by complete the steps to solve for x What is the factorization of 729x^15 + 1000? Suppose that theta is an angle in standard position whose terminal side Intersects the unit circle at (-11/61, -60/61)Find the exact values of tan theta, sec theta, and cos theta. A bicycle ramp used for competitions is a triangular prism. The volume of the ramp is 313.2 cubic feet. Write and solve an equation to find the width of the ramp. List and discuss how the nature of rural settlements affect the type and expanse of agricultural activities in the rural area please help with this for brainliest 5. A company is deciding which box to use for their merchandise. The first box measures 8 inches by 6.25 inches by 10.5 inches. The second box measures 9 inches by 5.5 inches by 11.75 inches. Which box required more material to make? 6. If each box (from #5) used material that cost $0.03 per square inch to make, how much does a company save by choosing to make fifty boxes of the smaller box in comparison to fifty boxes of the larger box? What is the value of y? how many whole numbers are there between 40 and 75 all meet at the North and South PolesA Longitude linesBylindrical projection approximations6. Mercator linesD. Latitude lines Brainliest + 25 POINTS!! HURRY please answer both questions! A can contains 414 mL of chicken broth. Roland poured 4 cans of broth into a pot for a soup recipe. How many liters of broth did Roland pour into the soup? Which defines a line segment?two rays with a common endpointa piece of a line with two endpointsa piece of a line with one endpointall points equidistant from a given pointB. A piece of a line with two endpoints During World War II, which of the following activities was considered patriotic?buying war bondscollecting rubber and scrap metalrationing goods, like butter and sugarAll of these choices are correct.